Difference between revisions of "2022 AMC 12A Problems/Problem 18"

(Redirected page to 2022 AMC 10A Problems/Problem 18)
(Tag: New redirect)
 
(6 intermediate revisions by 4 users not shown)
Line 1: Line 1:
==Problem==
+
#redirect [[2022 AMC 10A Problems/Problem 18]]
 
 
Let <math>T_k</math> be the transformation of the coordinate plane that first rotates the plane <math>k</math> degrees counter-clockwise around the origin and then reflects the plane across the <math>y</math>-axis. What is the least positive
 
integer <math>n</math> such that performing the sequence of transformations <math>T_1, T_2, T_3, \cdots, T_n</math> returns the point <math>(1,0)</math> back to itself?
 
 
 
==Solution==
 
Let <math>A_{n}</math> be the point <math>(\cos n^{\circ}, \sin n^{\circ})</math>.
 
 
 
Starting with <math>n=0</math>, the sequence goes <cmath>A_{0}\rightarrow A_{179}\rightarrow A_{359}\rightarrow A_{178}\rightarrow A_{358}\rightarrow A_{177}\rightarrow A_{357}\rightarrow\cdots</cmath>
 
 
 
We see that it takes <math>2</math> turns to downgrade the point by <math>1^{\circ}</math>. Since the fifth point in the sequence is <math>A_{177}</math>, the answer is <math>5+2(177)=\boxed{\textbf{(A)}~359}</math>
 
 
 
==Video Solution by Professor Chen Education Palace==
 
 
 
https://youtu.be/QQrsKTErJn8
 
 
 
==See also==
 
{{AMC12 box|year=2022|ab=A|num-b=17|num-a=19}}
 
{{AMC10 box|year=2022|ab=A|num-b=17|num-a=19}}
 
{{MAA Notice}}
 

Latest revision as of 05:17, 19 November 2022